The radius of a circle is 12.4 cm. Find the circumference to the nearest tenth.

Answers

Answer 1
77.9 cm should be the answer

Related Questions

A.54 pie cm^3

B.72 pie cm^3

C.126 pie cm^3

D.378 pie cm^3

Answers

Answer:  54pi  cm^3   (choice A)

==========================================================

Explanation:

The radius of each sphere is r = 3

The volume of one sphere is

V = (4/3)*pi*r^3

V = (4/3)*pi*3^3

V = 36pi

That's the volume of one sphere.

Three spheres take up 3*36pi = 108pi cm^3 of space.

---------------------------

The radius of the cylinder is also r = 3, since each tennis ball fits perfectly in the container.

The height is h = 18 because we have each ball with a diameter 6, which leads to the three of them stacking to 3*6 = 18.

The volume of the cylinder is...

V = pi*r^2*h

V = pi*3^2*18

V = 162pi

-------------------------

Subtract the volume of the cylinder and the combined volume of the spheres:  162pi - 108pi = (162-108)pi = 54pi

This is the exact volume of empty space inside the can.

This points to choice A as the final answer

What is the value of (-3 + 31) + (-2+31)?

Answers

Answer:

57

Step-by-step explanation:

31-3=28

31-2=29

28+29=57

-3+31-2+31= 57

Best of wishes!!

Is it true or false that for all sets A, B, and C, A U (B - C) = (A U B) - C?

Answers

Answer:yes

Step-by-step explanation:66

The given statement  A U (B - C) = (A U B) - C is true.

What is a set ?

A set is collection of well defined objects.

According to the given question we have to state whether A ∪ ( B - C ) = ( A ∪ B ) - C.

Lets consider we have three sets A, B and C and we also consider they intersect each other.

( B - C ) represents the elements which belongs to B but not in C.

∴ A ∪ ( B - C ) represents the no. of elements which belongs to the set B but not in C union the no. of elements which belongs to A.

AND

( A ∪ B ) - C represents no. of elements which belongs to A or B but not in C.

learn more about sets here :

https://brainly.com/question/8053622

#SPJ2

which equation is the inverse of 5y+4=(×+3)^2+1/2?​

Answers

Answer:

The inverse is -3 ±sqrt(5x+7/2)

Step-by-step explanation:

5y+4=(x+3)^2+1/2?

To find the inverse, exchange x and y

5x+4=(y+3)^2+1/2​

Solve for y

Subtract 1/2

5x+4 -1/2=(y+3)^2+1/2​-1/2

5x+8/2 -1/2=(y+3)^2+1/2​-1/2

5x+7/2 = (y+3)^2​

Take the square root of each side

±sqrt(5x+7/2) =sqrt( (y+3)^2​)

±sqrt(5x+7/2) = (y+3)

Subtract 3 from each side​

-3 ±sqrt(5x+7/2) = y+3-3

-3 ±sqrt(5x+7/2) = y

The inverse is -3 ±sqrt(5x+7/2)

Kay leaves a $6 dollar tip for a $60 dollar bill what percentage is the tip

Answers

The answer is (6/60)*100 = 10%

Answer: uhh 10%??

Step-by-step explanation:

I'm not rlly good at percentage-

there is 300ml of oil in the small bottle there is six times as much in the big bottle how much oil is in the big bottle?​

Answers

Answer:

1800 ml of oil

Step-by-step explanation:

300*6

Find the area for me pls

Answers

this figure can be divided into three parts one is rectangle other one is semicircle and the third one is one fourth of the circle .so let's find the area of each figure one by one. For the rectangle 12×8 =96

for semicircle that is on the top it has the radius 6 which is a half of 12

so area of the semicircle is

[tex] \frac{1}{2} \pi \: r {}^{2} \\ \frac{1}{2} \times 3.14 \times 6 {}^{2} \\ 3.14 \times 18 \\ 56.52[/tex]

no that's fine. 80 of the 1/4 of the other Circle

[tex] \frac{1}{4} \pi {}^{2} \\ \frac{1}{4} 3.14 \times 8 {}^{2} \\ 3.14 \times 16 \\ 50.24[/tex]

add all these areas

96+56.52+50.24 =202.76

PLEASE HELP!

Determine which of the following lists is in order from smallest to largest.

1. -3,131,0, (-3)^2
2. (-3)^2,-3,0, |3|
3. -3,0,|3|, (-3)^2
4. 0,-3,|3|, (-3)^2

Answers

Answer:

3. -3,0,|3|, (-3)^2

Step-by-step explanation:

Answer:

answer would be option 3

Step-by-step explanation:

help this helps

Area of a trapezoid
Find the area of this trapezoid. Be sure to include the correct unit in your answer.
6 cm
0
10 cm
8 cm
12 cm

Answers

Answer:

h

Step-by-step explanation:

h

the sum of two consecutive numbers is 2x+3. What are the numbers

Answers

Answer: 2 and 3

Step-by-step explanation:

its numbers

what is 5.73 as a mixed number​

Answers

Answer:

[tex]5 \ \frac{73}{100} [/tex]

Step-by-step explanation:

Convert the decimal number to a fraction by placing the decimal number over a power of ten. Since there are

2 numbers to the right of the decimal point, place the decimal number over 10^2 (100). Next, add the whole number to the left of the decimal.

Answer:

5 73/100

Step-by-step explanation:

5.73 = 573

100

= 573

100

as a fraction

To convert the decimal 5.73 to a fraction, just follow these steps:

Step 1: Write down the number as a fraction of one:

5.73 = 5.73

1

Step 2: Multiply both top and bottom by 10 for every number after the decimal point:

As we have 2 numbers after the decimal point, we multiply both numerator and denominator by 100. So,

5.73

1

= (5.73 × 100)

(1 × 100)

= 573

100

.

(This fraction is alread reduced,

As the numerator is greater than the denominator, we have an IMPROPER fraction, so we can also express it as a MIXED NUMBER, thus 573

100

is also equal to 5 73/100

when expressed as a mixed number.

The function f(x) = –x2 – 4x + 5 is shown on the graph. On a coordinate plane, a parabola opens down. It goes through (negative 5, 0), has a vertex at (negative 2, 9), and goes through (1, 0). Which statement about the function is true? The domain of the function is all real numbers less than or equal to −2. The domain of the function is all real numbers less than or equal to 9. The range of the function is all real numbers less than or equal to −2. The range of the function is all real numbers less than or equal to 9.

Answers

Answer:

D

Step-by-step explanation:

We have the quadratic function:

[tex]f(x)=-x^2-4x+5[/tex]

First, the domain of all quadratics is always all real numbers unless otherwise specified. You can let x be any number and the function will be defined.

So, we can eliminate choices A and B.

Note that since the leading coefficient is negative, the parabola will be curved downwards. Therefore, it will have a maximum value. This maximum value is determined by its vertex, which is (-2, 9).

Since it is curving downwards, the maximum value of the parabola is y = 9. It will never exceed this value. Therefore, the range or the set of y-value possible is always equal to or less than 9.

So, the range of the function is all real numbers less than or equal to 9.

Our answer is D.

It is not C because the maximum value is dependent on y and not x.

The relationship between the amounts of blue
and red paint needed to make a certain shade
of purple is shown in the following graph.
10+
9+
8+
7
6
Liters of red paint
5+
4+
3+
2
1

3
1
2
Liters of blue paint
Which statements about the graph are true?
Choose all answers that apply:

Answers

Answer:

B

Step-by-step explanation:

Had it on Khan Academy.

True statement for the given graph is the point [tex](3,9)[/tex] shows that [tex]9[/tex] liters of red paint are needed for every [tex]3[/tex] liters of blue paint.

What is graph?

" Graph is defined as diametrical representation of the relation between the variables on the coordinate plane along x-axis and y -axis."

According to the question,

As given in the graph,

Graph represents the linear relation between red paint and blue paint.

'x -axis' on the graph represents the blue paint

'y- axis' on the graph represents red paint

A. The point [tex](0,0)[/tex] shows that any amount of red and the blue paint will make same shade of purple.

From the graph coordinates [tex](1,3)[/tex] and [tex](2, 6)[/tex]represents that proportion of blue : red to make required purple shade is [tex]1: 3[/tex].

Hence, Option A is not a correct answer.

B.  The point [tex](3,9)[/tex] shows that [tex]9[/tex] liters of red paint are needed for every [tex]3[/tex] liters of blue paint.

Coordinate [tex]( 3,9)[/tex] on the graph represents the ratio of blue : red [tex]= 3 : 9[/tex] which is equals to [tex]1:3[/tex] .

Hence, Option(B) is the correct answer.

Learn more about graph here

https://brainly.com/question/20381610

#SPJ2

3 3/4 × 2 2/9 please
Help ‍♀️‍♀️‍♀️

Answers

[tex]\implies {\blue {\boxed {\boxed {\purple {\sf { \: 8 \frac{1}{3}\:(or) \:8.333}}}}}}[/tex]

[tex]\sf \bf {\boxed {\mathbb {Step-by-step\:explanation:}}}[/tex]

[tex]3 \frac{3}{4} \times 2 \frac{2}{9} [/tex]

➺[tex] \: \frac{15}{4} \times \frac{20}{9} [/tex]

➺[tex] \: \frac{300}{36} [/tex]

➺[tex] \: \frac{25}{3} [/tex]

➺[tex] \: 8 \frac{1}{3} [/tex]

➺[tex] \: 8.333[/tex]

[tex]\large\mathfrak{{\pmb{\underline{\pink{Mystique35 }}{\orange{❦}}}}}[/tex]

differentiate loge(x/x^2+7)​

Answers

Answer:

1+1=11 2+2=22 ok na yan kuya or ate

(View attachment)

a) Write ordered pairs.
b) Write the domain and range.
c) Why isn't the relation a function?
d) Which ordered pair should be removed to make the relation a function?

Answers

Answer:

in a relationship that maps elements from one set (the inputs) into elements from another set (the outputs), the usual notation for the ordered pairs is:

(x, y), where x is the input and y is the output.

In this case, the point where the arrow starts is the input, and where the arrow ends is the output.

a)

The ordered pairs are:

(28, 93)

(17, 126)

(52, 187)

(34, 108)

(34, 187)

b) The domain is the set of the inputs, in this case the domain is the set where all the arrows start, then the domain is:

{17, 28, 34, 52}

And the range is the set of the outputs, in this case the range is:

{93, 108, 126, 187}

c) A function is a relationship where the elements from the domain, the inputs, can be mapped into only one element from the range.

In this case, we can see that the input {34} is being mapped into two different outputs, then this is not a function.

d) We can remove one of the two ordered pairs where the input is {34},

So for example, we could remove:

(34, 108)

And then the relation would be a function.

Here is a table of values for y = f(x).
Х
-2 -1 0 1 2 3
4.
5
6
f(x) 5
6 7 8 9 10 11 12 13
Mark the statements that are true.

Answers

Step-by-step explanation:

the true answers are:

A. f(-1)=6

D. the domain for f(x) is the set

{-2,-1,0,1,2,3,4,5,6}

Which expression is equivalent to 4p^-4 10q -3? Assume ​

Answers

Step-by-step explanation:

Derive an expression for the equivalent width in a saturated line. Assume a Voigt profile, with the difference in optical depth between the center of the line and the wings being ~104. The wings of the line can be ignored. Define a frequency x1 = (v1 − v0)/ΔvD, where the optical depth τv = 1. Inside of x1 the line is fully saturated, and outside x1 the line is optically thin. Show that the equivalent width is

Note that the equivalent width is practically insensitive to the number density of absorbing material.

fill in the blanks the 2 digit largest whole number is______​

Answers

99 is the correct answer

If l=10, b=5, h=2, find the values of 2h(l+b)​

Answers

Answer:

60

Step-by-step explanation:

given:

l = 10

b = 5

h = 2

to find:

2h(l + b)

substitute the given values of l , b and h

=2*2(10 + 5)

=4*15

=60

[tex]\huge\textsf{Hey there!}[/tex]

[tex]\large\textsf{2h(l + b)}[/tex]

[tex]\large\text{If l = 10, b = 5, and h = 2, then substitute it into the equation!}[/tex]

[tex]\large\textsf{= 2(2)((10)+ 5)}[/tex]

[tex]\large\textsf{2(2) = \boxed{\bf 4}}[/tex]

[tex]\large\textsf{= 4(10 + 5)}[/tex]

[tex]\large\textsf{10 + 5 = \boxed{\bf 15}}[/tex]

[tex]\large\textsf{4(15)}[/tex]

[tex]\large\textsf{= \boxed{\bf 60}}[/tex]

[tex]\boxed{\boxed{\huge\text{Answer: \textst \bf 60}}}\huge\checkmark[/tex]

[tex]\large\text{Good luck on your assignment and enjoy your day!}[/tex]

~[tex]\frak{Amphitrite1040:)}[/tex]

How do I factor this equation fully?

Answers

3x(x^2+x-6)-40,

hope that answers your question and best of wishes!!

Consider the graph below: Point T(-2; 3) is a point on the Cartesian Plane such that B is the angle of inclination of OT. T(-2;3) у х 2.1 Calculate the following without the use of a calculator: a) tanſ b) 13 sin B.cosB (2)​

Answers

Answer:

(a) - 3/2

(b) - 78/25

Step-by-step explanation:

According to the trigonometry, the tangent of any angle is the ratio of rise to the run of the right angle triangle .

The sine of an angle is the ratio of rise to the hypotenuse of the right angle triangle.

The cosine of an angle is the ratio of run to the hypotenuse of the right angle triangle.

(a)

[tex]tan\beta = \frac{3}{-2} = \frac{-3}{2}[/tex]

(b)

[tex]13 sin\beta cos \beta = 13\times \frac{3}{\sqrt{3^2+2^2}}\times\frac{-2}{\sqrt{3^2+2^2}}\\\\13 sin\beta cos\beta = \frac{- 78}{25}[/tex]

Given f(x) = x2 + 24 - 12, find f(3)

Answers

Answer:

f(3) = 12

Step-by-step explanation:

f(3) = (3 x 2) + 24 - 12

     = 6 + 24 - 12

f(3) = 12

If this helps you, please give brainliest!

Answer:

Step-by-step explanation:

f(x) = [tex]x^{2}[/tex] + 24 - 12

substitute 3 inplace of x

f(3) = [tex]3^{2}[/tex] + 24 - 12

=9 + 24 - 12

21

If the question is f(x) = x2 + 24 - 12     {x*2 + 24 - 12}

now substitute 3 in place of x

f(3) = 3*2 + 24 - 12

=6 + 24 - 12

18

please help meeeee!!​

Answers

Step-by-step explanation:

[tex]\begin{aligned} -5x+4y &= 3\\\\ x&=2y-15 \end{aligned}[/tex]

The amount of increase from the cost to the selling price is called the?​

Answers

Answer:

profit

hope it is helpful to you

Step-by-step explanation:

Markup is the percentage amount by which the cost of a product is increased to arrive at the selling price.

please help show steps thx

Answers

Answer:

1) P = 282m, A = 141m^2

2) P = 82.4in, A = 40in^2

3) P = 62.7m, A = 73.1m^2

Step-by-step explanation:

1) top:L=12m, W=3m, mid: l= 12m, w= 12-7 = 5m, bot: l=15m, w=3m

Perimeter= 2(lw)

P = 2(12x3) + 2(12x5) + 2(15x3)

P = 2(36) + 2(60) + 2(45)

P = 72 + 120 + 90

P = 282m  

Area= lw

A = (12x3) + (12x5) + (15x3)

A = 36 + 60 + 45

A = 141m^2

2) Rectangle:l=7in, w=5in, Right Triangle:a=5-3=2in, b=12-7=5in

Perimeter= 2(lw) + (a+b+sqrt(a^2+b^2))

P = 2(7x5) + (2+5+sqrt(2^2+5^2))

P = 70 + 12.39

P = 82.4in

Area= lw + ((ab)/2)

A = (7x5) + ((2x5)/2)

A = 35 + 5

A = 40in^2

3) Semi-Circle: d=8m, r=8/2=4m, Right Triangle: a=8m, b=12m

Perimeter= pid + (a+b+sqrt(a^2+b^2))

P = 9pi + (8+12+sqrt(8^2+12^2))

P = 28.27 + 34.42

P = 62.7m

Area= (1/2)pir^2 + ((ab)/2)

A = (1/2)pir^2 + ((ab)/2)

A = (1/2)pi(4)^2 + ((8x12)/2)

A = 25.13 + 48

A = 73.1m^2

9x5
pls help meeeeeeeeee

Answers

Answer:

45

hope this helps

Answer:

45

Step-by-step explanation:

9x5=45

What is the exponential regression equation that fits these data? x у 1 4 2 9 3 27 4 90 5 250 6 580 O A. y = 39.16x2 - 169.38x+ 158.9 O B. y = 1.31(2.804) O C. y = 2.80(1.31") O D. y = 104.74x - 206.6​

Answers

Answer:

1.31(2.80)^x

Step-by-step explanation:

Given the data :

x : 1 2 3 4 5 6

y : 4 9 27 90 250 580

Using technology, the exponential regression data obtained by fitting the data is :

y = 1.31(2.80)^x

Where, y = predicted value

1.31 = Initial value

2.80 = growth fractor

x = period

Answer:

1.31(2.80)^x

trust me i just did the cst

Step-by-step explanation:

smallest to biggest pls

Answers

Step-by-step explanation:

45.399,45.454,45.545,45.933

Which angles are supplementary to 24? Select all that apply.​

Answers

Answer:

<2, <3, <6, <7

Step-by-step explanation:

Supplementary to <4 means that, <4 and that angle equals 180. So we will move angle by angle.

<1 = Is not supplementary because both are virtually opposite angle and virtually opposite angles are equal.

<2 = Is supplementary angle because it is a vertical linear pair with <4.

<3 = Is supplementary angle because it is horizontal linear pair with <4

<5 = Is not supplementary angle because they are alternate interior angles and alternate angles are equal.

<6 = Is supplementary because angles on the same side and same interior/exterior are supplementary. (By interior/exterior, I meant that they have to be either interior or exterior. No both)You can also use the reason that 6 and 2 are corresponding angles, and corresponding angles are equal. So, if 2 is supplementary to 4, then 6 is also supplementary to 4. I think the latter is better than the first.

<7 = Is supplementary because it is vertically opposite to 6 and 6 is supplementary, so 7 is also supplementary.

<8 = Not supplementary. Because 4 and 8 are corresponding angles and corresponding angles are not equal. And also 8 is equal to 5. And 5 is not supplementary to 4. That means 8 is also not.

Ugh... I really hope u understand, and don't think that I am blabbering.

Thank You

Please mark me as the brainliest

Other Questions
1. What is the maximum value of the linear density in a crystalline solid (linear density defined as the fraction of the line length occupied by atoms, assumed as spheres and only counted it their center is on the line)?2. What family of directions has the highest linear density in the FCC system? 3. What family of directions has the highest linear density in the BCC system? 4. What family of planes has the highest planar density in the FCC system? 5. What family of planes has the highest planar density in the BCC system? 6. What family of planes has the highest planar density in the HCP sytem? which elements in the following set are integers -8,3/4,-0.18,0,0.16,5,-2/7,6 Select all the correct images.Select the atoms that belong to the same element.16 protons 16 neutrons16 electrons11 protons 10 neutrons 11 electron13 protons 14 neutrons 13 electrons Select the correct answer. The numbers of pages in the books in a library follow a normal distribution. If the mean number of pages is 180 and the standard deviation is 30 pages, what can you conclude? A. About 60% of the books have fewer than 150 pages. B. About 16% of the books have fewer than 150 pages. C. About 95% of the books have more than 150 pages. D. About 16% of the books have more than 150 pages. classify the expression: 5x-4 I need help understanding these its my school work Use Pythagorean Theorem to find each missing length please help with the steps How could the council reduce the amount of litter in the nature reserve ? Ito ay pag-alis ng kontrol ng pamahalaansa ibat-ibang aspekto ng industriya at agrikultura upang mahikayat angmga dayuhang negosyante sa bansa. *, You are in the process of inoculating biochemical tests to identify a gram negative bacillus. So far, you have determined that it is oxidase +. Indicate the two most helpful biochemical tests that will enable you to identify this bacterial isolate. (check two)a. urease testb. SIM mediumc. phenol red glucose brothd. citrate agare. nitrate brothf. MR-VP broth Somebody please help me asap Is triangle XYZ = ABC ? If so, name the postulate that applies. A. Congruent - ASA B. Congruent - SAS C. Might not be congruent D. Congruent - SSS 6 Write 89.4945 correct to (a) nearest whole number, [1] (b) two decimal places. Find the perimeter of the figure below, in inches. The product of two integers is (-112).If one of them is (-8), find the other. Answer this please tLol HELP PLEASE MATH PROBLEM The following events took place in January 2018. Sports Equipment Rentals (SER) rents equipment on an hourly or daily basis to customers. SER prepares monthly financial statements. Match each event with the choice that correctly describes the effect of the transaction on the accounting equation. Increase/decrease means one asset account increases and another decreases by the same amount. Use this selection for the next eight questions: Assets Liabilities EquityA. Increase No effect Increase B. Increase Increase No effectC. No effect Decrease IncreaseD. Increase/Decrease No effect No effectE. No TransactionJanuary 3: SER purchases $2,000 of sports equipment on credit. January 8: Customers pay SER $8,500 for daily rentals for services provided over the past three days. January 16: SER receives $4,000 for a 2-week rental for equipment for several teams. The rental period begins on February 10, 2014. January 17: SER signs an agreement to provide $4,500 of equipment to a customer in early February. The customer has not yet made a payment. January 25: SER receives $3,000 for services provided and billed in the prior month. January 30: SER rents out skates for a party that day and bills the customer for $300.January 30: SER completes a contract by providing rental equipment to a private school from January 17-30. The school paid $500 for the rental in December 2000. January 31: SER receives $120 in interest on a note receivable. (SER loaned an employee $10, 000 last November and the employee is paying SER monthly interest. The employee will repay the $10, 000 principal after one year.) Baseball runner with a mass of 70kg, moving at 2.7m/s and collides head-on into a shortstop with a mass of 85kg and a velocity of 1.6m/s. What will be the resultant velocity of the system when they make contact with each other During the 1800's what was the political machines considered as